Paeds #1 Flashcards

1
Q

You are called acutely to the postnatal ward to see a 48-hour-old baby girl.
She was seen the day before by your colleague who performed her
baby-check and found no abnormalities. On arrival you find a baby who
is grunting and cyanotic. All pulses are palpable and there is no murmur
on auscultation of the chest. You transfer her to the neonatal unit where
you find her oxygen saturations are 55% in air.
Which of the following congenital cardiac lesions would be consistent with
these findings?

A. Coarctation of the aorta

B. Eisenmenger’s syndrome

C. Patent ductus arteriosus

D. Tetralogy of Fallot

E. Transposition of the great arteries

A

E – Transposition of the great arteries

This neonate is clearly unwell. With these features you should be very suspicious
of congenital sepsis or pneumonia so your priority would be to perform blood
cultures, CRP, FBC and chest X-ray. Empirical antibiotics (e.g. amoxicillin and
gentamicin) should be commenced without delay.
These features are also suspicious of a cardiac lesion. Tetralogy of Fallot would
usually not present so early in life and would usually be associated with a pulmonary
stenosis murmur. In transposition of the great arteries the great
vessels are reversed (transposed) with the aorta coming off the right ventricle
and the pulmonary artery off the left ventricle. Affected children are therefore
dependent on the ductus arteriosus to supply oxygenated blood to the systemic
circulation (‘duct dependent’). As the duct closes after birth the baby will
become profoundly cyanotic and acidotic. Chest X-ray shows a characteristic
narrow mediastinum with an egg-on-side appearance of the heart shadow.
The ‘switch operation’ (surgical swapping of the pulmonary artery and aorta)
is required as definitive management.
Coarctation of the aorta also often presents when the ductus closes but this is
not a cyanotic disease and the baby would usually present with shock, poor perfusion
of the limbs distal to the lesion and absent femoral pulses. Eisenmenger’s
syndrome is a cyanotic heart lesion that occurs later in life with reversal of flow
across septal defects due to right ventricular hypertrophy. A patent ductus arteriosus
presents with a loud machinery-like murmur in a child with heart failure.

How well did you know this?
1
Not at all
2
3
4
5
Perfectly
2
Q

A 12-year-old boy with a history of type-1 diabetes is seen in the emergency

department. His blood sugar is high and he is breathing much harder than
usual. An arterial blood gas is taken (on air):

pH 7.19
PaCO2 2.8 kPa
Base excess –9
PaO2 14.6 kPa

Normal reference ranges for arterial blood gases:

pH 7.36–7.44
PaCO2 4.7–6.0 kPa, 35–45 mmHg
Base excess +2
PaO2 .10.6 kPa, .80 mmHg (in air)

Which of the following is your interpretation of this blood gas?

A. Metabolic acidosis (uncompensated)

B. Metabolic acidosis with some respiratory compensation

C. Mixed metabolic and respiratory acidosis

D. Respiratory acidosis (uncompensated)

E. Respiratory acidosis with some metabolic compensation

A

B – Metabolic acidosis with some respiratory compensation

This boy is most likely in diabetic ketoacidosis (DKA) and has a metabolic
acidosis with respiratory compensation. However you do not have a result for
ketones therefore you cannot technically diagnose DKA. He will need aggressive
management as DKA is a life-threatening complication of type-1 diabetes.
Interpretation of blood gas results
pH (the power of hydrogen) is an inverse logarithmic scale. That is, rather than
following a linear pattern, each unit of pH is a 10-fold representation of Hþ
concentration. Therefore a shift from 7.5 to 6.5 is a 10-fold increase in Hþ
concentration. The body normally controls serum pH within a tight range. A
normal intracellular pH is required for the functioning of many enzyme
systems. When blood becomes profoundly acidotic (pH ,7) then cellular
function becomes impossible and death ensues.
An excellent method for interpreting blood gases can be found in the advanced
paediatric life support manual. Three questions should give you the interpretation
of any blood gas.
1. What is the pH?
Assess the pH. Is it acidotic or alkalotic? This is the overall status of the patient,
regardless of compensation.
2. What is the PaCO2
The Henderson-Hesselbach equation tells us that if you mix carbon dioxide
(CO2) with water, acid (Hþ) will be produced. CO2 is a good indicator of ‘ventilation’
and if the pH corresponds with the pCO2 then the anomaly is caused by
a respiratory problem (i.e. a high pCO2 with a low pH is a respiratory acidosis). If
the CO2 does not account for the pH then it is compensating for a metabolic
abnormality.
3. Confirm your findings by assessing the base excess/bicarbonate?
The base excess (i.e. an excess of base HCO3
2) indicates metabolic status. If the
base excess provides a cause for the abnormal pH then this is a metabolic picture
(i.e. a high pH and positive base excess is a metabolic alkalosis).
As a final tip always remember to assess the conditions of the blood gas, i.e.
what inspired oxygen concentration was the patient in, what type of sample
was it (arterial, venous, capillary), was it taken correctly and do the results fit
with the patient’s condition. Also, remember that you can have a ‘mixed’
blood gas, i.e. a mixed metabolic/respiratory acidosis where both the CO2
and the bicarbonate are low (a compensated respiratory acidosis would have
a high bicarbonate).

How well did you know this?
1
Not at all
2
3
4
5
Perfectly
3
Q

A 7-year-old girl is referred to the paediatrician as she has started showing
signs of puberty. On examination, she has some breast tissue and sparse
pubic hair, as well as axillary hair. She is growing well and appears to be
starting a growth spurt. Her mother is worried as no one else in her class
has any signs of puberty and she is being teased. She has not started to
menstruate yet. She has no other symptoms and there is nothing else of
note on examination.
What is the most likely cause of her precocious puberty?

A. Congenital adrenal hyperplasia

B. Hypothyroidism

C. Idiopathic

D. McCune–Albright syndrome

E. Pituitary tumour

A

C – Idiopathic

This girl has true precocious puberty and it is most likely to be idiopathic. Precocious
puberty is defined as the start of puberty in girls under the age of 8 and in
boys under the age of 10. It can be divided into true and pseudo.
In true precocious puberty the course of puberty occurs in a normal synchronous
manner, as in this case, suggesting an intact hypothalamic–pituitary
axis. It is more common in girls and is idiopathic in 80–90% of cases. Other
causes of true precocious puberty include: intracranial pathology such as
tumours (although pituitary tumours are more likely to be associated with
delayed puberty), haemorrhage, hydrocephalus, neurofibromatosis, cerebral
palsy and primary hypothyroidism. Although true precocious puberty is less
common in boys, if it does occur there is more likely to be a central cause and
therefore should be investigated. This would normally involve a brain MRI,
gonadotrophin levels and sex steroid levels.
In pseudo (false) precocious puberty the course of puberty occurs in an abnormal
manner. For example, axillary and pubic hair with a growth spurt is seen in
females but there is no breast budding. Boys may show features of puberty but
still have prepubertal-sized testes. Pseudo precocious puberty is gonadotrophinindependent
and usually extracranial. Causes include adrenal virilizing tumours,
congenital adrenal hyperplasia, Cushing’s syndrome, testicular or ovarian malignancy,
and gonadotrophin-secreting tumours, e.g. hepatoblastoma.
McCune–Albright syndrome is a sporadic genetic condition characterized by
precocious puberty (due to primary ovarian cysts secreting oestradiol), cafe´-
au-lait spots and polyostotic fibrous dysplasia (where normal bone is replaced
by cystic bone growth).

How well did you know this?
1
Not at all
2
3
4
5
Perfectly
4
Q

A 9-month-old boy has gastroenteritis. He is febrile and tachycardic. On
examination, he has a normal capillary refill time, a slight reduction in
skin turgor and dry mucous membranes. He is not wetting his nappies as
much as usual.
What percentage of weight do you expect to be lost from dehydration in
this case?

A. 0%

B. 5%

C. 10%

D. 15%

E. 20%

A

C – 10%

Approximately 70–80% of a child’s body is made up of water. Children with
fever should be assessed for signs of dehydration, such as1:
† Prolonged capillary refill time
† Abnormal skin turgor
† Abnormal respiratory pattern
† Weak pulse
† Cool extremities

How well did you know this?
1
Not at all
2
3
4
5
Perfectly
5
Q

A 3-year-old is in clinic and your consultant asks you to assess his
development. You find he demonstrates normal development in all areas.
Which of the following would you expect him not to be able to do?

A. Build a tower of five bricks

B. Copy a circle

C. Feed with a fork

D. Hop on one leg

E. Know his first and last name

A

D – Hop on one leg

A 3-year-old would be expected to be able to pedal a tricycle but not be able to
hop until they are 4 years old. In fine motor and vision development, a 3-yearold
should be able to copy a circle and build a tower of nine bricks. Languagewise
they would be speaking in at least three-word sentences and would know
their first and last names. They would not be expected to be able to count until
they were 4. Socially 3-year-olds play make-believe, can eat with a fork and can
brush their teeth. They should be being potty-trained and be dry during the day.

How well did you know this?
1
Not at all
2
3
4
5
Perfectly
6
Q

A 4-year-old boy presents to the emergency department with tachypnoea
and a fever. He has been vomiting and is passing only small amounts of
dark urine. On examination he is not clinically dehydrated and may even
be a little oedematous around his eyes. He is unable to tolerate oral antibiotics
so you insert a peripheral cannula. Because he is vomiting you
perform routine electrolyte studies while inserting the cannula. A chest
X-ray shows right lower lobe consolidation.

You receive the following blood results:

Na 129 (135–145 mmol/L)
K 4.2 (3.5–5.0 mmol/L)
Urea 3.2 (1.5–4.5 mmol/L)
Creatinine 83 (40–110 mmol/L)
pH 7.37 (7.35–7.45)
Glucose 4.2 (3.4–5.5 mmol/L)

Which of the following is the most likely cause of the electrolyte
disturbance?

A. Conn’s syndrome

B. Diabetes insipidus

C. Hypovolaemic hypernatraemia

D. Hypovolaemic hyponatraemia

E. Syndrome of inappropriate ADH secretion

A

E – Syndrome of inappropriate ADH secretion

This child has pneumonia associated with syndrome of inappropriate antidiuretic
hormone (SIADH). Though he is producing small volumes of concentrated
urine this is because of water retention rather than hypovolaemia. In
addition, if he was dehydrated, we may expect a raised urea.
A normal plasma sodium concentration is 135–145 mmol/L. Hyponatraemia is
usually defined as plasma Na ,130 mmol/L and hypernatraemia defined as
plasma Na .150 mmol/L.

How well did you know this?
1
Not at all
2
3
4
5
Perfectly
7
Q

You are at a wedding next to your 4-year-old nephew. He is eating a sausage
and suddenly starts coughing vigorously. He is very agitated and continues
to cough. He has been coughing for over a minute and he is now crying
between coughs.
You suspect an inhaled foreign body, which of the following would be your
next course of action?

A. Encourage coughing

B. Five back blows

C. Five chest thrusts

D. Heimlich manoeuvre

E. Rescue breaths

A

A – Encourage coughing

You may not become a paediatrician but an inhaled foreign body is relatively
common. As a general practitioner, parent or wedding guest you could instantly
become a hero! The vast majority of deaths occur in preschool-aged children.
Food is the most common object but almost anything can cause an obstruction.
If a foreign body is directly visible and accessible within the mouth then it can be
removed but great care has to be taken not to push it further into the airway.
Blind finger sweeps should never be performed.
If a child is coughing, such as in this case, then they should be encouraged as a
spontaneous cough is more likely to be effective than externally imposed
manoeuvres at relieving the obstruction. No further intervention should be
made unless the cough becomes ineffective. An effective breath is defined as
one in which the child is able to speak, cry or take breaths between coughs.
Though appearing much more distressing, it at least confirms a partially open
airway. Performing any kind of procedure may move the foreign object making
a partial obstruction a complete obstruction. If coughing then the child
should be continuously reassessed and never left alone

How well did you know this?
1
Not at all
2
3
4
5
Perfectly
8
Q

A 6-month-old girl born in India is referred to you by her general practitioner
as she is not growing along the centiles. She was born on the
50th centile and has now dropped below the 2nd centile despite a good
intake. She has had two chest infections but has never needed to be hospitalized.
On examination, she looks thin but is not pale. Her abdomen is not
distended and is soft on palpation.
What investigation is most likely to give you a cause for her failure
to thrive?

A. Full blood count

B. IgA

C. No investigation necessary

D. Sweat test

E. Thyroid function test

A

D – Sweat test

This girl is likely to have cystic fibrosis, which is diagnosed with a sweat test. She
was not born in the UK and so would have missed out on the neonatal screening.
A common presentation of cystic fibrosis is failure to thrive (FTT) along with
a history of chest infections.
FTT can be caused by numerous problems:
Non-organic FTT results from various environmental and psychosocial factors:
† Inadequate provision or intake of food
† Family dysfunction, lack of support and difficult parent–child interactions
† Children who are subject to neglect
Organic FTT:
† Prenatal causes
W Premature babies, intrauterine growth retardation
W Maternal infection
W Fetal alcohol syndrome
W Chromosomal abnormalities
† Inadequate intake
W Lack of appetite, seen in iron deficiency anaemia, and chronic infection
W Mechanical problems such as a poor suck or swallow secondary to
hypotonia
W Excessive vomiting caused by metabolic, renal or neurological disease,
or gastroesophageal reflux leading to oesophagitis can lead to food
refusal
† Poor absorption or utilization of metabolites
W Cystic fibrosis due to the lack of pancreatic enzymes
W Coeliac disease causes malabsorption but is unlikely in this case as there
are no other signs such as a distended abdomen or diarrhoea
W Hypothyroidism, diabetes mellitus, inborn errors of metabolism, renal
failure
† Increased metabolic demand
W Hyperthyroidism, congenital heart disease, chronic lung disease, renal
failure
W Malignancy
FTT must be distinguished from constitutionally delayed growth, a variation of
normal growth. Children with short stature resulting from constitutional delay
have a deceleration of growth in the first 2 years that can be confused with
FTT, but then grow parallel to but below the third percentile. In reality children
with failure to thrive would have more than one investigation and you would
most likely include a full blood count, thyroid function tests and a coeliac
screen. However in this case a sweat test is most likely to reveal the cause as
there are no other signs of hypothyroidism, coeliac disease or iron deficiency
anaemia.

How well did you know this?
1
Not at all
2
3
4
5
Perfectly
9
Q

A 28-year-old woman has asked the community midwife if it is appropriate
for her to breastfeed.
Which of the following is a contraindication to breastfeeding?

A. Cosmetic breast surgery

B. Mastitis

C. Maternal digoxin therapy

D. Maternal HIV in a developed country

E. Maternal HIV in a developing country

A

D – Maternal HIV in a developed country

The contraindications to breastfeeding are:
† Breast cancer
† Maternal infection
† HIV in developed country
† Active tuberculosis
† Hepatitis B
† Hepatitis C
† Active herpes infection of the breast
† Infantile galactosaemia
† Maternal medication, e.g. lithium, antimetabolite chemotherapies (not
digoxin)
† Maternal intravenous drug user
Unicef states that ‘Without preventive interventions, approximately one-third of
infants born to HIV-positive mothers contract HIV through mother-to-child
transmission, becoming infected during their mothers’ pregnancy, childbirth
or breastfeeding. The risk of HIV-infection has to be compared with the risk of
morbidity and mortality due to not breastfeeding. Breastfeeding is protective
against death from diarrhoea, respiratory and other infections, particularly in
the first months of life.’ It is for this reason that breastfeeding is recommended
in developing countries despite maternal HIV infection.
A physician needs to understand the benefits of breastfeeding in order to decide
if the benefits of breastfeeding are outweighed by the risks.
The advantages of breastfeeding to the baby are a reduced risk of infections (as
breast milk provides secretory IgA, lactoferrin, peroxidases and lysozymes),
reduced risk of sudden infant death syndrome and less diarrhoeal illnesses.
Advantages to the mother include the fact it is free, it aids the bonding
process, it provides a contraceptive effect (due to high prolactin levels) and
there is a reduced risk of premenopausal breast and ovarian cancers. Babies
fed with artificial feeds are more at risk of necrotizing enterocolitis, urinary
tract infections, gastrointestinal infections, respiratory infections, ear infections,
asthma, eczema and type-1 diabetes

How well did you know this?
1
Not at all
2
3
4
5
Perfectly
10
Q

A 12-year-old boy presents to the emergency department with worsening
shortness of breath and a wheeze. You suspect an acute exacerbation of
asthma.
Which of the following peak expiratory flow rates (best or predicted) would
suggest a severe exacerbation of asthma?

A. 75–100%

B. 50–75%

C. 33–50%

D. ,33%

E. ,20%

A

C – 33–50%

The peak expiratory flow rate (PEF) is often used to help guide diagnosis and
management of asthma. The levels of severity of acute exacerbations of
asthma have been defined by the British Thoracic Society, as follows:

Near fatal asthma Raised PaCO2 and/or requiring mechanical
ventilation with raised inflation pressures

Life-threatening asthma Anyone of the following in a patient with
severe asthma:
† PEF ,33% best or predicted
† SpO2 ,92%
† PaO2 ,8 kPa
† Normal PaCO2 (4.6–6.0 kPa)
† Silent chest
† Cyanosis
† Feeble respiratory effort
† Bradycardia
† Dysrhythmia
† Hypotension
† Exhaustion
† Confusion
† Coma
Severe asthma Any one of:
† PEF 33–50% of best or predicted
† Tachypnoea (dependent on age)
† Tachycardia (dependent on age)
† Inability to complete sentences in one
breath

Moderate asthma † Increasing symptoms
† PEF 50–75% of best or predicted
† No features of acute severe asthma

How well did you know this?
1
Not at all
2
3
4
5
Perfectly
11
Q

A 4-year-old boy, who was previously fit and well, developed some petechial
spots on his legs after climbing a tree. A few days later, the rash had spread
over his entire body. He had not been any more lethargic than normal and
was eating well. On examination, you note a large bruise on his hip and a
black eye which his mother could not give any explanation for.
Which of the following investigation would be likely to confirm the
diagnosis?

A. Bone marrow biopsy

B. Clotting screen

C. Full blood count

D. Skeletal survey

E. No investigation required

A

C – Full blood count

This boy most likely has idiopathic thrombocytopenia (ITP) which is diagnosed
by a very low platelet count (,40 109/L) with no evidence of anaemia and a
normal white cell count. Unexplained or extensive bruising in any child is
worrying.
Causes of bruising include:
† Trauma (accidental or non-accidental)
† Clotting disorders
† Platelet disorders
† Malignancy
Non-accidental injury cannot be fully ruled out in this case as the bruises are not
accounted for and a thorough history would need to be taken. It is unusual to
have a widespread petechial rash with physical abuse, although one may be
seen over the face and neck from strangulation. A diagnosis of non-accidental
injury should certainly not be made until other causes had been ruled out.
Leukaemia and other malignancies can present with unexplained bruising,
however the child in this case is very well and there are no other symptoms
that would suggest an underlying malignancy.

How well did you know this?
1
Not at all
2
3
4
5
Perfectly
12
Q

A 5-year-old girl has suffered from arthritis for the last 6 months. It only
affects her knees and elbows, and she has never had sacroiliac tenderness
or nail problems. She has no rash or fever, but she does have regular ophthalmology
follow-up due to her increase risk of developing uveitis.
Blood tests reveal that she is antinuclear antibody positive but rheumatoid
factor negative.
What type of arthritis is she most likely to have?

A. Enthesitis

B. Oligoarticular juvenile idiopathic arthritis

C. Polyarticular juvenile idiopathic arthritis

D. Psoriatic arthritis

E. Systemic juvenile idiopathic arthritis

A

B – Oligoarticular juvenile idiopathic arthritis

Juvenile idiopathic arthritis (JIA) is defined as arthritis of unknown aetiology
beginning before the 16th birthday and persisting for at least 6 weeks where
other known conditions are excluded. JIA is categorized into:
† Systemic JIA
† Oligoarticular JIA
† Polyarticular JIA
† Psoriatic arthritis
† Enthesitis-related arthritis
Oligoarthritis affects four joints or fewer, accounts for 60% of all JIA and is seen
most commonly in girls below 6 years. It affects the medium-sized joints such as
the elbows and knees, and is associated with anterior uveitis (inflammation of
the middle eye) as well as having the highest association with the antinuclear
antibody (ANA).
Polyarticular JIA affects five or more joints and often affects the small joints of the
hands and feet. This girl is unlikely to have polyarticular arthritis as it is seen more
commonly in older girls and rheumatoid factor is often positive. The cervical
spine and temporomandibular joints can also be affected.

How well did you know this?
1
Not at all
2
3
4
5
Perfectly
13
Q

An 8-year-old boy presents to the emergency department with abdominal
pain, fever, nausea and some diarrhoea. The pain is poorly localized,
although on palpation you feel he is most tender in the right lower quadrant
and he demonstrates guarding over this area. There are no obvious
swellings.
What is the most likely cause for his abdominal pain?

A. Appendicitis

B. Gastroenteritis

C. Inguinal hernia

D. Mesenteric adenitis

E. Torsion of the testis

A

A – Appendicitis

Appendicitis is common in children and is caused by inflammation and swelling
of the appendix. The lumen to the appendix becomes blocked and the appendiceal
wall subsequently becomes oedematous and inflamed. Initially, the pain is
poorly defined and periumbilical, but moves to the right iliac fossa (RIF) due to
inflammation of the peritoneum over the appendix. Nausea, anorexia, vomiting
and a low grade fever may also be present. The child with appendicitis typically
lies still and there is usually tenderness and guarding over the McBurney’s point
(one-third of the way between the anterior superior iliac spine and the umbilicus).
There are a number of other signs suggesting peritoneal irritation in
the RIF:
Rovsing’s sign ! pain in the RIF in response to left-sided palpation
Cough sign ! pain in the RIF after a voluntary cough
Obturator sign ! pain on internal rotation of the flexed right thigh
caused by an inflammatory mass overlying the
psoas muscle
Perforation is a complication of appendicitis and is more common in younger
children because of the non-classical presentation and late diagnosis. Appendicitis
can present differently to the symptoms given above. The pain may be elsewhere
in the abdomen or, in children who are very young, abdominal pain is
often not obvious. Loose stools occur in 10% of cases due to a pelvic or retrocaecal
appendix. Urinary symptoms may also be present if the appendix is in
the pelvis. Dysuria occurs in 10% of cases with an excess of leucocytes in the
urine.

How well did you know this?
1
Not at all
2
3
4
5
Perfectly
14
Q

A 3-year-old girl presents to the emergency department with a 3-day history
of diarrhoea and vomiting. Her mother is worried as she has blood in her
stool. On examination, she appears mildly dehydrated and pale but is
otherwise well. You perform some blood tests and a urine dipstick.

Bloods:
Hb 7.2 g/dL
WCC 8.0 g/dL
Platelets 69  109/L
Na 145 mmol/L
K 6.1 mmol/L
Urea 32 mmol/L
Creatinine 219 mmol/L
Urine dipstick:
Protein 3þ
Blood Negative
Leucocytes Negative
Nitrites Negative

What is the most likely diagnosis?

A. Glomerulonephritis

B. Haemolytic uraemic syndrome

C. Henoch–Scho¨nlein purpura

D. Leukaemia

E. Pyelonephritis

A

B – Haemolytic uraemic syndrome

This girl is anaemic, thrombocytopenic and is in acute renal failure. The most
likely diagnosis is haemolytic uraemic syndrome (HUS).
HUS is characterized by acute renal failure, microangiopathic anaemia and
thrombocytopenia (with a normal clotting time). It is commonly seen after gastroenteritis
caused by Escherichia coli (the verotoxin-producing O157:H7 strain),
Salmonella, Shigella or Campylobacter. Children present with vomiting and diarrhoea
(which is often bloody) and acute renal failure occurs soon after. This form
of HUS is known as the epidemic/typical version and is seen more commonly in
younger children and in the summer months. It usually has a good outcome and
permanent renal damage is uncommon. However the rarer type (‘sporadic
type’) is seen in older children and renal damage is more severe. Drugs and
malignancy can also all cause HUS, and there is a hereditary form. A blood
film is diagnostic, showing a microangiopathic haemolytic anaemia.
Complications of HUS include abdominal pain, myocarditis, encephalitis-like
features, hepatitis, pancreatitis and retinal haemorrhages. Renal damage can
lead to hypertension. Treatment is conservative and symptomatic, and includes
careful fluid and electrolyte balance, treatment of renal failure with dialysis and
transfusion of blood and platelets as required.
Henoch–Scho¨nlein purpura (HSP) is a vasculitis which affects the whole body. It
classically presents with a purpuric rash over the back of the legs and buttocks. It
is not associated with anaemia or thrombocytopenia but can lead to nephritis
and thus haematuria and proteinuria. Arthritis is a common feature and HSP
can also affect the abdomen, leading to abdominal pain and occasionally
melaena. Glomerulonephritis may present in a number of ways: nephrotic or
nephritic syndrome, acute renal failure or asymptomatic haematuria and/or proteinuria.
It does not normally present with anaemia and thrombocytopenia.
Leukaemia is unlikely in this situation as, although the child has anaemia and
thrombocytopenia, she is otherwise well. Acute renal failure can be a presenting
feature of malignancies if there is tumour lysis syndrome, but it is unlikely in a
child that is otherwise well.

How well did you know this?
1
Not at all
2
3
4
5
Perfectly
15
Q

A newborn baby has a heel-prick blood test taken for Guthrie card
screening.
Which of the following is not screened for on the newborn Guthrie card?

A. Congenital hypothyroidism

B. Cystic fibrosis

C. Diabetes mellitus

D. Phenylketonuria

E. Sickle cell disease

A

C – Diabetes mellitus

The national newborn screening programme was introduced in 1969 for phenylketonuria
(PKU) and in 1981 congenital hypothyroidism (CHT) was added.
Currently all babies in the UK will have a Guthrie card to screen for:
† PKU
† CHT
† Cystic fibrosis
† Sickle cell disease and thalassaemia
Medium Chain Acetyl Co-A Dehydrogenase Deficiency (MCADD – an inborn
error of metabolism where fat cannot be broken down, resulting in low
energy and hypoglycaemic episodes) screening has now also been approved
and will hopefully soon be nationalized.
The Wilson criteria describe the features of a good screening tool:
† The condition should be an important health problem
† The natural history should be understood
† There should be a recognizable latent or early symptomatic stage
† There should be an accepted treatment recognized for the disease
† Treatment should be more effective if started early
† There should be a policy on who should be treated
† Diagnosis and treatment should be cost-effective
The test/tool should be:
† Easy to perform and interpret
† Acceptable
† Accurate, reliable, sensitive and specific
† Case-finding should be a continuous process

How well did you know this?
1
Not at all
2
3
4
5
Perfectly
16
Q

A 10-year-old boy falls from a lower branch of a tree while playing with his
brother at home. There was no loss of consciousness at the time but he now
has a headache and is feeling sick. He is brought to the emergency
department 20 minutes later. On examination there is no focal neurology.
Which of the following is an indication for an immediate CT scan?

A. Amnesia of the event and the preceding 4 hours

B. Glasgow Coma Score of 14 on admission

C. Obvious tender swelling to the side of the head

D. One episode of vomiting in the department

E. Previous head injury 2 years previously

A

A – Amnesia of the event and the preceding 4 hours

Initial assessment of any head injury follows A-B-C. Cervical spine evaluation is
performed in conjunction with airway assessment. C-spine immobilization is
appropriate if any of the following apply: Glasgow Coma Scale (GCS) ,15,
neck tenderness or pain, any focal neurological deficit, paraesthesia or other
clinical suspicion of a cervical spine injury. Immobilization should be maintained
until a full risk assessment including clinical assessment (and imaging if necessary)
indicates removal is safe.
If GCS ,8, an anaesthetist should secure the airway by means of tracheal intubation
and ventilation. A full skeletal survey should also be performed to assess
any other injuries.

Indications for CT head scanning in patients under 16 years are as follows:
† Witnessed loss of consciousness lasting .5 minutes
† Amnesia (anterograde or retrograde) lasting .5 minutes
† Abnormal drowsiness
† Three or more discrete episodes of vomiting
† Clinical suspicion of non-accidental injury
† Post-traumatic seizure but no history of epilepsy
† Age .1 year: GCS ,14 on assessment in the emergency department
† Age ,1 year: GCS (paediatric) ,15 on assessment in the emergency
department
† Suspicion of open or depressed skull injury or tense fontanelle
† Sign of basal skull fracture
W Haemotympanum (blood behind the tympanic membrane)
W Raccoon eyes
W Cerebrospinal fluid otorrhoea/rhinorrhoea
W Battle’s sign (blood at the mastoid)
† Focal neurological deficit
† Age ,1 year: presence of bruise, swelling or laceration .5 cm on the head
† Dangerous mechanism of injury (high-speed road traffic accident either as
a pedestrian, cyclist or vehicle occupant, fall from .3 m, high-speed
injury)

How well did you know this?
1
Not at all
2
3
4
5
Perfectly
17
Q

A 13-month-old boy presents to the general practitioner. He has a 24-hour
history of irritability. His mother reports he has been ‘tugging’ at his left ear.
His temperature is 38.28C. On examination of the left ear, there is a bulging
red tympanic membrane.
Which of the following describes the appropriate management of this case?

A. Five-day course of oral antibiotics

B. Insertion of oil to the external ear canal

C. No action required

D. Oral analgesia

E. Oral antihistamine

A

D – Oral analgesia

Acute otitis media (AOM) is a purulent middle ear process. Earache is the single
most important symptom. Other ear-related symptoms include tugging and
rubbing of the ear, irritability, restless sleep and fever. Non-specific symptoms
such as cough and rhinorrhoea may also be present. Examination with an otoscope
may reveal a bulging tympanic membrane with loss of the normal landmarks,
a change in colour (red or yellow) and poor mobility.
In this case it would be entirely reasonable to use paracetamol for control of discomfort
and as an antipyretic.
The following guidance is given by the Scottish Intercollegiate Guidelines
Network regarding the management of AOM1:
† Children with AOM should not routinely be prescribed antibiotics as the
initial treatment
† If an antibiotic is to be prescribed, the conventional 5-day course is recommended
at dosage levels indicated in the British National Formulary
† Delayed antibiotic treatment (antibiotic to be collected at parents discretion
after 72 hours if the child has not improved) is an alternative approach
which can be applied in general practice
† Parents should give paracetamol for analgesia and be advised of the
potential danger of overuse
† Children with AOM should not be prescribed decongestants or
antihistamines
† Insertion of oils should not be prescribed for reducing pain in children with
AOM

How well did you know this?
1
Not at all
2
3
4
5
Perfectly
18
Q

A 4-year-old boy presents with a 10-month history of cough. This occurs
most nights and he is usually symptom-free during the day. He has had
two episodes of wheeze, the first at 9 months of age associated with bronchiolitis,
the second when he was 3 years old when he had a viral upper respiratory
tract infection.
Which of the following would you like to perform?

A. Chest X-ray

B. Peak expiratory flow rate before and after
bronchodilator

C. pH study

D. Sweat test

E. Trial of therapeutic bronchodilators

A

E – Trial of therapeutic bronchodilators

Asthma is the most common chronic respiratory disease in children. It is defined
as ‘a chronic inflammatory disorder of the airways in susceptible individuals.
Inflammatory symptoms are usually associated with widespread but variable
airflow obstruction and an increase in airway response to a variety of stimuli.
Obstruction is often reversible, either spontaneously or with treatment’.
Chronic cough is a common problem in paediatrics and general practice. The
differential diagnosis includes:
† Asthma (cough exacerbated by exercise, frequent nocturnal cough, atopic
family history)
† Gastroesophageal reflux (associated with feeding reflux symptoms)
† Inhaled foreign body
† Inhaled irritants
† Allergic rhinitis (sneezing, itchy eyes, watery rhinorrhoea)
† Sinusitis (prolonged nasal discharge)
† Post-infectious cough syndrome
† Habit cough (a cough not responsive to treatment which only occurs
during waking hours)
† Tuberculosis
† Cystic fibrosis (now picked up more frequently on Guthrie card and so
many do not develop symptoms before the diagnosis is made)
† Immune deficiency (frequent upper respiratory tract infections)
† Bronchiectasis
Asthma is the most common cause of recurrent cough in children. It is usually
associated with wheeze and/or breathlessness, and triggered by exercise, cold
or dust. In many children a nocturnal cough may be the only presenting clinical
feature.

How well did you know this?
1
Not at all
2
3
4
5
Perfectly
19
Q

An 8-year-old girl has recently been diagnosed with type-1 diabetes.
Her current insulin regimen is 10 units of Mixtard 30 in the morning and
7 units of Mixtard 30 in the evening. Her recent blood glucose monitoring
is shown below:
Time of day Blood glucose
Pre-breakfast 5 mmol/L
Pre-lunch 10 mmol/L
Pre-tea 19 mmol/L
Pre-bedtime 8 mmol/L
How should she change her insulin regimen?

A. Change evening dose to 7 units Mixtard 20

B. Change morning dose to 11 units Mixtard 20

C. Increase evening insulin to 8 units Mixtard 30

D. Increase morning insulin to 11 units Mixtard 30

E. Increase morning insulin to 11 units Mixtard 30 and evening dose
to 8 units Mixtard 30

A

D – Increase morning insulin to 11 units Mixtard 30

This girl is on a twice daily insulin regimen. The insulin she takes is a mixture of a
short-acting and medium/long-acting insulin. For example, Mixtard 30 contains
30% short-acting insulin and 70% medium-acting insulin. When a child is
initially diagnosed an estimate of 0.5–1.0 units/kg/day insulin is often used to
start with. This is usually split 2/3 at breakfast time and 1/3 at teatime. Monitoring
of diabetes is done by measuring blood sugars daily. Preprandial (before
food) blood glucose should aim to be between 4 and 8 mmol/L and postprandial
to be 10 mmol/L. When morning glucose levels are high this suggests the
medium-acting evening insulin needs to be increased. If pre-lunch glucose
levels are high the short-acting breakfast dose needs to be increased.
It is best to only adjust one insulin dose at any time and allow a few days to assess
the outcome. It is also useful to check blood sugar more regularly when changing
insulin regimens. The amount to increase or decrease the dose by varies,
but a rough estimate of 10% either way is often used.
In this case the girl’s glucose level is high pre-lunch and pre-tea, though the prelunch
level is not excessively high. Her morning blood sugars however appear
well controlled. This would imply she needs her morning medium-acting
insulin increased. You could do this by either increasing the Mixtard 30 dose
or by increasing the amount of medium-acting insulin in her morning dose,
i.e. change her to Mixtard 20 (20% short acting and 80% long acting).
However it is best not to adjust the dose and the type of insulin together as it
is difficult to know which was effective. Therefore sticking with Mixtard 30
but increasing the morning dose is the most optimal management in this case

How well did you know this?
1
Not at all
2
3
4
5
Perfectly
20
Q

A 5-year-old boy presents to the emergency department with a red, nonblanching
rash over his trunk and peripheries. His mother says that he
has been complaining of a headache. He is crying but is limp in his
mother’s arms. He is cool to touch and has a central capillary refill time
of 4 seconds.
The child has an oxygen mask before you arrive. Which of the following
would be your first course of action?

A. Give a bolus of antibiotic

B. Give a fluid bolus of 10 mL/kg

C. Give a fluid bolus of 20 mL/kg

D. Intubation and ventilation

E. Perform a lumbar puncture

A

C – Give a fluid bolus of 20 mL/kg

In children under 3 years of age the classical signs of neck rigidity, photophobia,
headache and vomiting are often absent making clinical diagnosis of meningitis
harder.

Signs of possible meningitis in children under 3 years are:
† Coma
† Drowsiness
† High-pitched cry or irritability
† Poor feeding
† Unexplained pyrexia
† Convulsions with or without fever
† Apnoea or cyanotic attacks
† Purpuric rash
A purpuric rash in an ill child with a headache should raise a strong suspicion of
meningitis associated with sepsis. The most common organism responsible for
causing septicaemia in infants and children is meningococcus (Neisseria meningitidis).
Group B streptococcus is also common in infants. The Advanced Paediatric
Life Support manual (the gold standard for acute care) states that all children
who are acutely unwell should be approached in an ABC fashion. This includes
meningitis. In this case the patient is crying therefore we know that the airway is
intact and the child is breathing. It would be useful to assess the respiratory rate,
monitor blood saturations and apply some facial oxygen to minimize tissue
hypoxia. Though intubation and ventilation may become necessary it is not
required at this point.
Capillary refill time (CRT) is a measure of perfusion. It is performed by applying
cutaneous pressure on the centre of the sternum (central) or digit (peripheral)
for 5 seconds. The ‘refill’ or return of colour should occur within 2 to 3
seconds. CRT is a very useful sign in early septic shock when the child may be
otherwise well. In this case, as the perfusion is poor, your first action would be
to site a peripheral cannula (taking a blood sugar, blood culture, full blood
count, C-reactive protein, venous blood gas and polymerase chain reaction
for organisms) and then give a fluid bolus (20 mL/kg is the recommended
first bolus). This is to improve the perfusion to maximize the distribution of antibiotics
which should be given immediately after the fluid bolus. Antibiotics
should not be delayed for a lumbar puncture

How well did you know this?
1
Not at all
2
3
4
5
Perfectly
21
Q

A 4-month-old boy presents to the emergency department with vomiting,
poor feeding and excessive crying for the last 8 hours. He has previously
been a well boy with normal development and growth, but was born prematurely.
On examination, he looks unwell and appears in pain particularly
when you press in the lower right quadrant. He is chubby but you think
you can feel a mass in his groin.
What is the most likely diagnosis?

A. Appendicitis

B. Inguinal hernia

C. Intussusception

D. Pyloric stenosis

E. Volvulus

A

B – Inguinal hernia

This child presents with an inguinal hernia. Inguinal hernias in children are most
commonly indirect (pass down the inguinal canal into the scrotum, lateral to the
inferior epigastric vessels) and are the result of a patent processus vaginalis.
Indirect inguinal hernias have an incidence of 5% and are more common in premature
babies, males and on the right-hand side (due to delayed descent of the
right testis). Children often present with an intermittent swelling in the groin
which is more prominent after crying. An incarcerated hernia (this case) presents
with poor feeding, vomiting, crying and a painful lump. Incarcerated hernias are
a medical emergency as they can quickly lead to bowel strangulation and perforation.
All uncomplicated inguinal hernias should have an elective herniorrhaphy
to prevent incarceration.

Pyloric stenosis is due to hypertrophy and hyperplasia of the muscular layers of
the pylorus, causing a functional gastric outlet obstruction. It usually presents
between 3 and 8 weeks of age with non-bilious vomiting which becomes projectile.
Affected babies are extremely hungry but have poor weight gain and
may appear dehydrated. Pyloric stenosis is diagnosed by giving the baby a
‘test feed’: when the baby is given milk, visible gastric peristalsis may be seen
over the epigastrium and the pylorus is felt as an olive-shaped mass in the
upper abdomen, If the diagnosis is in doubt an ultrasound can be performed.
Investigations reveal a hypochloraemic, hypokalaemic metabolic alkalosis.
Infants often need to be rehydrated before definite surgical treatment (Ramstedt’s
pyloromyotomy – the muscle of the pylorus is cut longitudinally down
to the mucosa). The baby can tolerate milk feeds a few hours after the operation.
Intussusception describes the invagination of a proximal portion of bowel (the
intussusceptum) into a distal segment (the intussuscipiens), most commonly
at the ileocaecal junction. This telescoping of the bowel leads to bowel wall
oedema, ischaemia and eventually necrosis. It is most common between 6
and 18 months of age. The typical presentation is of a previously healthy
baby experiencing bouts of colicky abdominal pain and vomiting. There is
often diarrhoea which eventually looks like ‘redcurrant jelly’ (engorgement
and ischemia of the intestinal mucosa cause bleeding and an outpouring of
mucus). A sausage-shaped mass may be palpable in the right upper quadrant.
Management is by resuscitation followed by reduction of the intussusception
by a rectal air enema. If this is ineffective (25% of cases) then operative reduction
is required.
Appendicitis presents classically with poorly defined periumbilical pain which
shifts to the right lower quadrant due to the inflammation of the peritoneum
over the appendix. A mass is not normally felt, though nausea, vomiting and
a low grade fever are associated.
Volvulus is defined as the complete twisting of a loop of bowel around its mesenteric
stalk. It is associated with malrotation of the bowel and most commonly
presents in the neonatal period, but can be seen in children up to 12 months
old. The hallmark features are bilious vomiting and abdominal pain, but in
older children symptoms may be vague (chronic intermittent vomiting,
abdominal pain, constipation and failure to thrive).

22
Q

You review a baby boy on the postnatal ward as the midwives are worried
the child is hypotonic. The baby is indeed hypotonic and you also notice
he has an unusual looking face. He has a flat nasal bridge, almond-shaped
eyes with prominent epicanthic folds and low set ears. His genitalia are
normal.
What is the most likely cause of this child’s signs?

A. Down’s syndrome

B. Fetal alcohol syndrome

C. Hypothyroidism

D. Prader–Willi syndrome

E. Turner syndrome

A

A – Down’s syndrome

Down’s syndrome (trisomy 21) is estimated at 1 per 800 to 1000 births. As
maternal age increases so does the incidence of Down’s (1:385 risk at 35
years, 1:30 at 45 years). However more children with Down’s syndrome are
born to younger mothers. The triple test is used to give a risk to the mother
and not a diagnosis. Amniocentesis or chorionic villus sampling needs to be
done for a definite diagnosis.

There are numerous physical features of Down’s syndrome and not all children
will have all the signs. There is usually brachycephaly (short head) and a short
neck. In the face upslanting palpebral fissures, a flat nasal bridge, low set ears
and almond-shaped eyes caused by an epicanthic fold on the medial aspect
of the eye, and Brushfield spots (white spots on the iris) are seen. The hands
of children with Down’s syndrome have a single palmar crease (in 50% of
cases), a high number of ulnar loop dermatoglyphs (fingerprint loop patterns
that start on the ulnar side), clinodactyly (incurving of the little finger) and a
single flexion furrow of the fifth finger. A sandal gap (large space between the
big and second toes) is seen in the feet. Hypotonia is often seen as well as
short limbs and excessive joint laxity.
Nearly all children with Down’s syndrome have some form of learning disability
ranging from mild (IQ 50–70) to moderate (IQ 35–50). A small percentage of
children have severe to profound learning difficulties. Children with Down’s syndrome
have an increase risk of certain medical conditions, including congenital
heart defects (atrioventricular septal defect [AVSD] and ventricular septal defect
[VSD]), gastroesophageal reflux, hypothyroidism, cataracts and strabismus,
recurrent otitis media, obstructive sleep apnoea, leukaemia, atresias (especially
duodenal) and Hirschsprung’s disease.
Hypotonia is seen in hypothyroidism but affected children usually have coarse
facial features. In Prader–Willi, hypotonia and almond-shaped eyes are also
seen but there is often cryptorchidism and genital hypoplasia. Babies with
fetal alcohol syndrome are not particularly hypotonic and have characteristic
facies with a thin top lip, short nose and small eyes with hypertelorism
(increased distance between the eyes). Turner syndrome is seen only in girls
and is not associated with hypotonia.

23
Q

A 3-year-old girl has increased tone of her lower and upper limbs. Her legs
are affected more than her arms and she has exaggerated reflexes in both
her legs. You note that she has an abnormal gait, walking on her tiptoes
with her knees and hips both flexed. She was born at 25 weeks gestation
and she had a stormy course during the neonatal period.
What type of cerebral palsy is this girl most likely to have?

A. Ataxic cerebral palsy

B. Athetoid cerebral palsy

C. Diplegic cerebral palsy

D. Hemiplegic cerebral palsy

E. Quadriplegic cerebral palsy

A

C – Diplegic cerebral palsy

Cerebral palsy (CP) describes a group of disorders affecting movement caused
by a permanent, non-progressive lesion in the developing brain. It is primarily
a disorder of movement, but is often coupled with other neurological problems.
Although it is non-progressive, the clinical picture can change as the child grows
and develops. Diagnosis is clinical. During the first year of life there is abnormal
tone (initial hypotonia which eventually leads to spasticity). There is usually
delay in gross motor development or abnormal movements. Often the primitive
reflexes, such as the Moro and grasp reflex, persist longer than they should.
Associated problems include learning difficulties (ranging from mild to
severe), epilepsy, visual impairment, hearing loss, speech disorders, behavioural
disorders and respiratory problems.

Cerebral palsy can be divided into different forms:
† Spastic (70%)
W Hemiplegic
W Diplegic
W Quadriplegic
† Ataxic
† Dyskinetic
† Mixed
The girl in this scenario most likely has diplegia as both her legs and arms are
involved but the upper limbs are less severely affected. These children often
demonstrate scissoring of the legs due to excessive adduction of the hips, and
have a characteristic gait: the feet are equinovarus (plantarflexed and turned
inward), the hips and knees are flexed, and they walk on tiptoes. This form of
CP is seen in association with periventricular leukomalacia (white-matter
injury near the cerebral ventricles, seen on ultrasound or MRI) often found in
ex-premature babies. Affected children often do not have severe learning difficulties
or epilepsy.
Hemiplegia affects one side of the body only and it is the arm that is often more
affected than the leg. They have a typical gait which is accentuated when
running – the affected leg is extended at the knee and circumduction occurs
(the foot needs to make a semicircle with the toe scraping the ground in
order to move forward); the arm is also in a typical position (elbow and wrist
flexed with finger extension). Quadriplegia is the most severe form of CP,
where all limbs are significantly involved. It is often associated with learning difficulties,
epilepsy, swallowing problems and gastroesophageal reflux. Affected
children often develop flexion contractures by late childhood.

24
Q

A 4-year-old girl is referred to the paediatric clinic due to poor growth. She
weighed 3.5 kg at birth and initially grew along the 50th centile for weight,
length and head circumference. Since her first birthday her growth has
tailed off and her height is now well below the 0.4th centile with her
weight on the 9th centile. The only other history of note was she was
admitted to the neonatal unit when she was born because of low blood
sugars and jaundice.
What investigation is most likely to reveal her underlying problem?

A. Bone age

B. Chromosomal analysis

C. Growth hormone test

D. Mid-parental centile

E. Thyroid function tests

A

C – Growth hormone test

Growth velocity is far more important than an isolated measurement of height.
This child is most likely to have growth hormone deficiency. This leads to short
stature (about half the normal growth velocity) associated with a markedly
delayed bone age. There is a normal rate of growth until 6–12 months of
age, then the growth velocity tails off. It is also associated with neonatal hypoglycaemia,
jaundice and a doll-like face. Hypothyroidism is also associated with
short stature but there is often weight gain and other signs of thyroid disease
such as dry skin, constipation and bradycardia. Routine Guthrie testing assesses
for thyroid function and would rule out the majority of cases of hypothyroidism.
In familial short stature, the child will grow along the centiles, there will be no
delay in bone age and the parental heights will be low. It is therefore important
to know the parents’ heights and calculate the mid-parental height. This child is
not growing along the centiles and therefore is unlikely to have familial short
stature.

25
Q

You see a 7-year-old boy in the emergency department who has a history of
asthma. He says he had a cold yesterday and now presents with a 6-hour
history of ‘feeling tight’. On arriving he tells you his name is Jonny and
that he’s disappointed to be here as he was going to see Leeds United this
afternoon. On examination, he has equal air entry bilaterally associated
with a loud polyphonic wheeze. His saturations are 93% in air and his
peak expiratory flow rate is 60% of his best in clinic. His heart rate is 110/
min and his respiratory rate is 28/min.
Which of the following would be your most immediate step?

A. Give a b-agonist via a nebulizer

B. Give a b-agonist via a spacer

C. Give oral steroids

D. Start oxygen via a face mask

E. Take no action

A

B – Give a b-agonist via a spacer

The symptoms may not sound bad, but this is by definition a moderate acute
exacerbation of asthma. Do not treat it lightly! Moderate attacks can soon
become severe if adequate bronchodilation is not established rapidly. The
British Thoracic Society guidelines are the gold standard for managing
asthma, covering diagnosis through to management. In this case using a
b-agonist (such as salbutamol) should help open up the airways. This boy will
also need a course of oral steroids, but that would not be your first step. It is difficult
to know how to administer b-agonists during an acute exacerbation;
especially as using a nebulizer is practically a lot easier. However, a recent
Cochrane review did show that children using a spacer would spend less time
in A&E than if a nebulizer was used1.

26
Q

A term baby is born by elective Caesarean section as the baby is in a breech
presentation. The mother has been well during the pregnancy and a vaginal
swab from a previous pregnancy was clear. A few hours after birth you are
called to see the baby as he is ‘working hard’. On examination he has a respiratory
rate of 100/min, is afebrile and well perfused. The chest is clear
with equal air entry.
Which of the following is the most likely diagnosis?

A. Congenital pneumonia

B. Pneumothorax

C. Respiratory distress syndrome

D. Sepsis

E. Transient tachypnoea of the newborn

A

E – Transient tachypnoea of the newborn

Transient tachypnoea of the newborn (TTN) is due to a delayed clearance of
fluid from the fetal lungs. This makes TTN much more common in babies
born by elective Caesarean section. During vaginal delivery, adrenaline levels
rise in the baby due to stress which causes active uptake of fetal lung fluid
via sodium channels. As there is no stress during an elective Caesarean, the
adrenaline-mediated response does not occur, leaving fluid in the newborn’s
lungs. Babies with TTN can have respiratory rates as high as 100 to 120/min.
A chest X-ray should be performed to rule out congenital pneumonia. In TTN
the chest X-ray will show hyperinflation, oedema and fluid in the fissures.
Most infants with TTN will not require any treatment other than oxygen.
It is not possible to differentiate TTN from early-onset sepsis, so empirical antibiotics
should be started until infection has been entirely ruled out. There are
no risks for sepsis in this scenario (negative swab, no prolonged rupture of membranes)
so sepsis unlikely.
Maternal factors that increase the risk of sepsis and congenital pneumonia
include:
† Prolonged rupture of membranes (.18 hours)
† Maternal group B streptococcus on high vaginal swab or urine
† Maternal fever (.388C)
† In utero fetal tachycardia .160/min
† Maternal antibiotics (for suspected sepsis)
Respiratory distress syndrome is caused by a deficiency of surfactant in the lungs
and primarily occurs in preterm infants.

27
Q

A 2-year-old girl presents with a 6-week history of foul smelling diarrhoea.
She has also lost some weight. On examination, her abdomen looks full and
slightly distended.
Which of the following is the most appropriate first-line investigation that
would identify the cause?

A. Antigliadin IgA

B. Large bowel biopsy

C. Small bowel biopsy

D. Stool sample

E. Tissue transglutaminase IgA

A

E – Tissue transglutaminase IgA

This girl has coeliac disease, an autoimmune gluten-related enteropathy of the
small bowel. When the patient consumes gluten (a protein found in wheat, rye
and barley) a T-cell-mediated autoimmune reaction occurs, causing damage to
the intestinal mucosa. It affects 1 in 1000 people and children can present with failure to thrive, diarrhoea, steatorrhoea, fatigue, abdominal distension and
pain, muscle wasting and iron deficiency anaemia.
Antigliadin antibody was initially used in diagnosing coeliac disease. This has
now been replaced with tissue transglutaminase (tTGA) and anti-endomyseal
(EMA) IgA testing which have been shown to have sensitivities and specificities
of over 95%. Guidelines at present recommend using IgA tTGA as the initial
screening test and, if this is positive, to add EMA to further increase the specificity.
It must be remembered that those who are IgA-deficient may have a
falsely negative test and therefore should have IgA levels measured at the
same time as the tTGA. The gold standard diagnostic technique is duodenal
or jejunal biopsy taken during endoscopy. Characteristic histological findings
in coeliac disease are:
† Partial to total villous atrophy
† Crypt hyperplasia and lengthening
† Intraepithelial lymphocytes
Other causes of villous atrophy include temporary gluten intolerance secondary
to gastroschisis, kwashiorkor, tropical sprue, cows’ milk intolerance, giardiasis
and severe combined immunodeficiency.
Coeliac disease is treated with a strict lifelong gluten-free diet. There is an
increase risk of enteropathy-associated T-cell lymphoma and non-Hodgkin’s
lymphoma, as well as an association with other autoimmune diseases such as
vitiligo and thyroid disease. Dermatitis herpetiformis (an intensely itchy, blistering
papulovesicular rash found on the extensor surfaces) is associated with coeliac
disease and may respond to gluten-avoidance or dapsone.

28
Q

A 4-year-old boy is brought to the general practitioner with a cold. On
auscultating the chest, a loud murmur is heard. The murmur occurs at
the beginning of systole and is heard loudest at the upper left sternal
edge. It radiates to the back and is associated with a thrill.
Which of the following is the most likely diagnosis?

A. Aortic stenosis

B. Coarctation of the aorta

C. Mitral regurgitation

D. Pulmonary stenosis

E. Ventricular septal defect

A

D – Pulmonary stenosis

Heart murmurs are a relatively common finding in children and many pathological
lesions have distinct patterns of murmurs. Murmurs are caused by turbulent
blood flow, usually through valves but also through vessels and heart chambers.
Most murmurs are innocent such as vibratory murmurs and venous hums. Innocent
murmurs are generally asymptomatic, systolic, are louder during fever and
exercise, and vary with respiration and posture.
Heart murmurs are graded on their intensity:
Grade I Faint murmur, usually only cardiologists can hear
Grade II Quiet murmur
Grade III Moderate intensity, no thrill
Grade IV Loud murmur, with a thrill
Grade V Very loud murmur with a thrill, still requires a stethoscope to
hear
Grade VI Very loud murmur with thrill, can be heard close to chest wall
without a stethoscope

29
Q

An 18-month-old girl presents to the general practitioner with fever and a
runny nose. On examination, there are no signs of a serious illness, though
she has a temperature of 388C. The mother is concerned as she had a febrile
convulsion 4 months ago.
Which of the following statements is true regarding this febrile child?

A. Do not routinely give antipyretics to solely reduce body
temperature

B. Give ibuprofen and paracetamol simultaneously to maximize effect

C. Strip the child to her nappies

D. Tepid sponging is recommended

E. Use antipyretics to prevent febrile convulsions

A

A – Do not routinely give antipyretics to solely reduce body
temperature

The NICE guidelines for feverish illness in young children recommend the
following for antipyretic interventions:
† Tepid sponging is not recommended
† Do not over- or under-dress a child with fever
† Consider either paracetamol or ibuprofen as an option if the child appears
distressed or is unwell
† Take the views and wishes of parents and carers into account when considering
the use of antipyretic agents
† Do not routinely give antipyretic drugs to a child with fever with the sole
aim of reducing body temperature
† Do not administer paracetamol and ibuprofen at the same time, but
consider using the alternative agent if the child does not respond to the
first drug
† Do not use antipyretic agents with the sole aim of preventing febrile
convulsions
A traffic light system has been recommended for the identification of acutely
unwell febrile children. This is based on assessments of colour, activity, respiratory
function, hydration and other signs of serious illness (see NICE guidelines1).

30
Q

A 14-year-old girl is referred to paediatric outpatient as she has not entered
puberty yet. She has no breast development or pubic hair. On examination,
she is well and a full neurological examination is normal. She is short for
her age but has no dysmorphic features. Her mother says that she herself
‘developed late’ but cannot remember exactly when she went through
puberty.
Which investigation should be done initially?

A. Gonadotrophin levels

B. Karyotyping

C. Ovarian ultrasound

D. Pituitary CT

E. No investigation required

A

A – Gonadotrophin levels

Delayed puberty is defined as the absence of pubertal development by the age
of 13 in females and 14 in males, or the failure of progression over a 2-year
period. Pubertal delay is seen more commonly in boys but is more likely
to have an underlying cause in a girl. This girl warrants investigation. Gonadotrophins are a useful first-line investigation to determine which further
investigations are needed, as delayed puberty can either be associated with
low or high gonadotrophin secretion.
The most common cause of delayed puberty in boys is constitutional delay and
there is often associated growth delay and a family history of pubertal delay.
However this is a diagnosis of exclusion and there are other important causes
of delayed puberty which need to be ruled out, especially in girls. Examination
should include pubertal staging, accurate height and weight and a bone age.
Constitutional delay causes pubertal delay secondary to low gonadotrophin
(leuteinizing hormone and follicle-stimulating hormone) secretion. Chronic
illness such as cystic fibrosis, asthma, Crohn’s and anorexia can also lead to
low gonadotrophin secretion. Other causes include disorders of the hypothalamic–
pituitary axis including hypothyroidism, panhypopituitarism, Kallmann
syndrome (inability to smell [anosmia], decrease in gonadotrophin-releasing
hormone, developmental delay) and intracranial tumours. Visual field examination
should be performed and, where there is suspicion of a pituitary
tumour, a CT needs to be performed. Thyroid and pituitary function tests
help assess the function of the hypothalamic–pituitary axis.
Raised gonadotrophin levels with defective gonads can also lead to delayed
puberty. This is seen in Turner syndrome (XO) and Klinefelter’s syndrome
(XXY). If examination is suspicious, karyotyping (chromosomal analysis)
should be requested. Damage to the gonads can also be secondary to chemotherapy
or radiotherapy, postsurgery or torsion. Imaging of the gonads
can be performed when there is doubt.

31
Q

You are called to the emergency department where a 4-year-old boy is
having a seizure. He is known to have epilepsy and is on regular phenytoin.
He was brought in by ambulance and the paramedic has given buccal
midazolam en route. The emergency doctor has cited a cannula from
which a sample of blood has revealed a blood sugar of 4.8 mmol/L. He is
still fitting 10 minutes after the buccal midazolam.
Which of the following would be your next step in management?

A. Bolus 10% dextrose

B. Intravenous lorazepam

C. Intravenous phenytoin

D. Intubation and ventilation

E. Rectal paraldehyde

A

B – Intravenous lorazepam

Managing a seizure is an acute emergency. Benzodiazepines are the backbone
of seizure management and will be successful in the termination of most seizures
if given correctly and promptly. Manage all seizures in an ABC (DEFG – do not
ever forget glucose) fashion. Managing the airway and assessing breathing is
paramount as many seizures will terminate themselves and one needs to only
ensure adequate oxygenation. If the seizure continues, one member of the
team should continue to assess and manage the airway, breathing and
circulation.
Check the blood sugar as hypoglycaemia is a common and easily managed
cause of convulsions (using 5 mL/kg 10% dextrose).
If a patient has intravenous access, the order of management is:
1. ABC (DEFG)
2. Lorazepam intravenously (2 doses)
3. Paraldehyde rectually. Draw up and give phenytoin intravenously while
giving the paraldehyde (unless already on phenytoin in which case give
phenobarbitone)
4. Intubation and ventilation with thiopental
This algorithm does not cover all cases as some children with documented epilepsy
may respond to certain drugs and so often an individual protocol can be
more appropriate. One would usually wait 10 minutes between each step;
however preparing yourself with the next medication would be common practice.
During this period the child should have their airway maintained while
monitoring breathing and circulation. This is particularly important once
benzodiazepines have been given as they can cause respiratory depression.
An anaesthetist should be called early if you suspect that intubation and ventilation
are likely.

32
Q

You are working in the emergency department when you are called to an
ambulance. A mother has just delivered her baby a few seconds before
arriving. As you enter the ambulance the paramedic is clamping the cord.
The baby is not breathing and looks floppy and pale.
Which of the following would be your most immediate action?

A. Administer five rescue breaths

B. Cardiac compressions

C. Evaluate breathing

D. Manage airway

E. Warm and dry the baby

A

E – Warm and dry the baby

The resuscitation council have given guidance on the best steps to take during
resuscitation of a newborn. The resuscitation of a newborn is the only emergency
of which we are aware that requires a step before ‘airway’. Warming
and drying the baby is vitally important as cold babies are more likely to
develop acidosis, hypoglycaemia and reduced surfactant production.
In practice, warming and drying the baby, opening the airway and assessing the
newborn all form one swift movement for an experienced paediatrician. The
steps in neonatal resuscitation can be summarized as:
† Clamp the cord
† Warm and dry the baby
† Open and clear the airway
† Evaluate breathing, heart rate, colour and tone
† Five inflation breaths
† Continuing ventilation
† Cardiac compressions
In this case, once the baby has been warmed and dried, it is likely it will need
active resuscitation as it is not breathing and is pale and floppy. Opening the
airway and aerating the lungs is usually successful in most cases. This is done
by placing the head in the neutral position (not head-tilt–chin-lift) and giving
five inflation breaths.

33
Q

An 8-year-old boy presents with a 3-hour history of a painful, swollen testicle.
The pain started gradually overnight and he now scores it as 6/10. He
has no nausea and has not vomited. On examination, his abdomen is soft
but there is an obvious swelling and redness of the left scrotum. There is
mild tenderness in the upper area of the testicle. His cremasteric reflex is
present.
What is the most likely diagnosis?

A. Epididymitis

B. Hydrocele

C. Inguinal hernia

D. Torsion of the hydatid of Morgagni

E. Torsion of the testicle

A

D – Torsion of the hydatid of Morgagni

This boy is most likely to have torsion of the hydatid of Morgagni (a small embryological
remnant at the upper pole of the testes) which is the most common cause of an acute scrotum in children. It is a benign condition but needs to be
distinguished from testicular torsion, which is a surgical emergency.
Torsion of testes refers to the twisting of the spermatic cord in or just below the
inguinal canal. Strangulation of the gonadal vessels can lead to testicular necrosis
and atrophy. Testicular torsion is more common in adolescents whereas
torsion of the hydatid is seen in younger children, typically 7 to 12 years. The
pain in torsion of the hydatid is usually less severe than testicular torsion and
can have a more insidious onset (as in this case). Both cause a red swollen
testis, but in torsion of the hydatid the tenderness is usually localized to the
upper pole of the testes, in contrast to the extremely tender testicle, found
high in the scrotum, in testicular torsion. The pain in testicular torsion is also
often referred to the abdomen. Other distinguishing features of testicular
torsion include nausea and vomiting. The cremasteric reflex is preserved in
torsion of the hydatid and there may be a ‘blue dot’ (the torted hydatid)
visible on the scrotum when it is transilluminated.
Irreversible infarction of a twisted testicle occurs within 6–12 hours, so affected
patients should be taken to theatre for surgical exploration without further
investigation. (Generally, to preserve spermatogenesis, torsion should be
relieved within 4 hours.) Surgical management includes untwisting of the testicle
and bilateral fixation of the testes to the tunica vaginalis to prevent further
torsion. Bilateral fixation is required because anatomical abnormalities of the
testes that predispose to torsion usually occur on both sides.
A hydrocele presents with a swollen scrotum but is usually soft and non-tender. It
is difficult to palpate the testicle as it lies within the fluid collection. Epididymitis,
the inflammation of the epididymis, presents with a swollen painful testicle.
There is often an associated fever as well as nausea. It can be difficult to distinguish
from testicular torsion but Prehn’s sign is indicative of epididymitis (tenderness
relieved by elevating the scrotum). There may also be associated urinary
symptoms including frequency and dysuria.

34
Q

A 3-year-old girl is brought to her general practitioner. She has been treated
for a chest infection for 10 days with antibiotics but is still no better. Her
mum has noticed that she is pale and lethargic. On examination, the girl
appears unwell and the doctor is able to palpate the spleen. He also
notices a number of bruises over her arms and body which mum is
unable to account for.
What is the most likely diagnosis?

A. Acute lymphoblastic leukaemia

B. Acute myeloid leukaemia

C. Cystic fibrosis

D. Glandular fever

E. Neglect and physical abuse

A

A – Acute lymphoblastic leukaemia

This child is most likely to have leukaemia with a combination of splenomegaly,
bruises, lethargy and pallor. Acute lymphoblastic leukaemia (ALL) accounts for
80% of childhood leukaemias and is therefore the most likely answer. ALL has
a peak incidence at around 5 years and slowly decreases in adolescence. It is
slightly more common in boys and a higher incidence is seen in Caucasians.
Acute myeloid leukaemia (AML) is not nearly as common in childhood, though it
accounts for 80% of all acute leukaemias when including adults. There is an
increase incidence of leukaemia in Down’s syndrome with an equal incidence
of AML and ALL. Bloom’s syndrome (an autosomal recessive syndrome characterized by short stature, facial photosensitive rash, narrow face, subfertility
and immune deficiency) and neurofibromatosis are also associated with an
increased risk.
Features of leukaemia depend on the degree of infiltration into the bone marrow
and other sites in the body. These range from an incidental finding on full blood
count in a well child to life-threatening situations where the child is in tumour
lysis syndrome. The symptoms are related to firstly the direct infiltration of the
bone marrow by leukaemic cells (bone and muscle pain) or direct infiltration
of other organs (hepatosplenomegaly or lymphadenopathy). Other symptoms
are related to the decreased production of normal marrow elements including
white cells, haemoglobin and platelets. Anaemia leads to pallor, lethargy, dizziness
and exertional dyspnoea. The white cell count may be normal, low or high
but the neutrophils are nearly always low. This leads to an increase in infections
and the inability to recover from seemingly mild infections. Platelets are often
low, resulting in bruising, epistaxis, bleeding gums and petechiae. Occasionally,
if the leukaemia is already in the central nervous system, children may present
with nausea, vomiting or even a focal neurological deficit.
Cystic fibrosis is often picked up on neonatal screening (Guthrie card) though it
can present with failure to thrive and recurrent chest infections. Splenomegaly
can be seen in cystic fibrosis, secondary to liver disease, but is unlikely to be a
presenting sign. Neglect and physical abuse can present with failure to thrive
and bruising but in an acutely unwell child with splenomegaly this is unlikely
to be the cause. Glandular fever, caused by Epstein–Barr virus, can present
with lethargy and prolonged fever. However, pharyngitis is the typical presentation
not a chest infection. Splenomegaly may be present, but bruising is not
a feature.

35
Q

A 9-year-old boy suffers from epilepsy. His mother describes his seizures as
infrequent. She normally hears a strange noise in the night from her son’s
room and then finds him twitching the right-hand side of his mouth and
face. He is usually drooling and is unrousable. This lasts for a few minutes
and the child has no recollection of the event. A subsequent EEG shows
high amplitude spikes in the left centrotemporal region.
What is the most likely type of epilepsy this boy has?

A. Absence seizures

B. Benign rolandic epilepsy

C. Juvenile myoclonic epilepsy

D. Lennox–Gastaut type epilepsy

E. Tonic–clonic epilepsy

A

B – Benign rolandic epilepsy

This child has a typical history of benign rolandic epilepsy which accounts for
20% of all childhood epilepsy. Seven to 10-year-olds are most likely to be
affected (but it can be seen from 3 to 13 years of age) and there is a male preponderance.
Rolandic seizures are usually nocturnal involving the mouth and
face. They often begin with an odd sensation at the corner of the mouth
which leads to twitching of the mouth and then the rest of the ipsilateral
face. Excessive salivation, grunting and slurred speech can occur, and they
can progress to generalized seizures. The EEG often shows high amplitude
spikes in the left centrotemporal region. This area of the brain is near the
motor strip (the Rolandic fissure, hence the name of the epilepsy). It is a
benign condition and children often grow out of it by adolescence.
Juvenile myoclonic epilepsy usually begins around puberty and can be precipitated
by alcohol. There are usually severe symmetrical jerks of the arms and
trunk, which can proceed to generalized fits. They often occur in the morning
on waking.
Absence seizures are generalized seizures affecting consciousness which typically
last a few seconds. The child stares blankly and is unresponsive. They are often noticed in school and can be confused with daydreaming. There is no involuntary
movement or falling to the ground, but when children become alert after
the seizure they have no recollection of the event. Absence seizures occur
between the ages of 2 and 10 years, are more common in girls and children frequently
grow out of them. The EEG typically shows three spike-wave complex
cycles per second. There is a family history in 20% of cases.
Lennox–Gastaut type epilepsy is rare and presents between 1 and 4 years of age.
Features include daily seizures, episodes of status epilepticus, slowed psychomotor
development and behavioural disorders. It has a poor prognosis. The EEG
shows slow spike-waves with multiple abnormalities.

36
Q

A 2-month-old boy is brought to the general practitioner by his mother for
routine immunizations.
Which of the following immunizations should not routinely be given at
2 months of age?

A. Diphtheria

B. Haemophilus influenzae

C. Meningitis C

D. Pneumococcus

E. Tetanus

A

C – Meningitis C

Immunizations and the immunization schedule commonly appear in written
papers. Much more commonly though is an objective structured clinical examination
(OSCE) station in which you will need to counsel a parent about the
need for immunizations. As part of this you should have a basic understanding
of which immunizations are given at certain ages.
The current immunization programme (2010) is described below:
† 2 months ! DTaP/IPV/HiB, pneumococcal
† 3 months ! DTaP/IPV/HiB, meningitis C (MenC)
† 4 months ! DTaP/IPV/HiB, pneumococcal, MenC
† 12 months ! HiB/MenC
† 13 months ! Measles, mumps and rubella (MMR), pneumococcal
† Preschool ! DTaP, IPV and MMR
† 13–18 years ! DTaP

37
Q

A 5-month-old infant was found to be failing to thrive. On examination, a
quiet murmur is heard at the lower left sternal edge throughout systole.
Over the past 2 months he has become short of breath during feeding.
His weight has now fallen down two centile lines.
Which of the following would be your first step in management?

A. ACE inhibitor

B. Digoxin

C. Diuretics

D. Insertion of a feeding nasogastric tube

E. Surgical correction

A

D – Insertion of a feeding nasogastric tube

This child has a ventricular septal defect (VSD). The history also suggests that he
is developing heart failure due to the extra work of the left side of the heart supplying
the systemic circulation as well as blood flowing to the right heart via the
septal defect. This boy has a quiet (grade I) murmur. Remember that quiet murmurs often signify a large lesion as there is little turbulent flow. A small
defect on the other hand usually causes a loud murmur due to the highly turbulent
flow across the septum.
Heart failure can be caused by structural lesions (e.g. coarctation, VSD, AVSD,
large PDA) or by the failure of a structurally normal heart (e.g. supraventricular
tachycardia, myocarditis, cardiomyopathy). The signs of heart failure are breathlessness,
poor feeding, sweating and recurrent chest infections. On examination,
you may find a child who is failing to thrive, tachypnoeic and
tachycardic with a murmur, hepatomegaly or a displaced apex due to cardiomegaly.
The definition of heart failure is the inability of the heart to keep up with the
metabolic requirements of the tissues of the body. The management of heart
failure starts with supportive steps to maximize the provision of metabolic
requirements to the body. Therefore inserting a nasogastric tube and starting
high caloric feeds will reduce the associated growth retardation. Growth retardation
is caused by a failure to take adequate feeds due to breathlessness and
secondly due to a high metabolic rate in the symptomatic child. Commencing
oxygen therapy will also aid in the provision of oxygen to the tissues.
It is likely that this child will need pharmacological management. Diuretics (thiazide
or loop) will reduce the load on the heart. ACE inhibitors are frequently
used in conjunction with diuretics. There is now little evidence for the use of
digoxin. Surgery is the definitive treatment and is usually delayed until the
child is big enough to tolerate the procedure. Some children need to have surgical
correction in the first year of life; they include babies with severe heart
failure and failure to thrive, or pulmonary hypertension with the potential to
progress to pulmonary vascular disease.

38
Q

An anxious mother attends the general practice as her 7-day-old newborn
has sticky eyes. Both eyes have a mucopurulent exudate. You take swabs
including a scraping and advise the mother to clean the eyes four times a
day with cooled, boiled tap water. Three days later you are informed that
an intracellular organism has grown from the samples you sent.
Which of the following organisms is responsible for the infection?

A. Chlamydia trachomatis

B. Neisseria gonorrhoeae

C. Pseudomonas aeruginosa

D. Staphylococcus aureus

E. Streptococcus pneumoniae

A

A – Chlamydia trachomatis

Ophthalmia neonatorum is the term used for conjunctivitis occurring in the first
few weeks of life. A standard swab for microscopy and culture should be performed
along with a conjunctival scraping. The aim of the scraping is to gain
cells within which chlamydial organisms can be found.
The most common causes of ophthalmia neonatorum are:
† Staphylococcus aureus
† Chlamydia trachomatis
† Neisseria gonorrhoeae
† Chemical irritation (silver nitrate)
Chlamydia is now amongst the most common causes of neonatal conjunctivitis.
C. trachomatis is an intracellular organism and is treated with tetracycline eye
drops and oral erythromycin for 2 weeks. The other organisms here are not
intracellular and therefore cannot be responsible. C. trachomatis can also
cause pneumonia and this should be considered if the child develops any respiratory
symptoms.

39
Q

A 3-year-old boy presents to the general practitioner with a 2-month history
of cough. It started with a cold, but that has since resolved. Currently, he
has severe bouts of coughing, occasionally followed by vomiting. On examination,
he has bilateral equal air entry and no crepitations.
Which of the following is the most likely diagnosis?

A. Asthma

B. Bronchiolitis

C. Inhaled foreign body

D. Pneumonia

E. Whooping cough

A

E – Whooping cough

Whooping cough is caused by the bacterium Bordetella pertussis. It is now less
common in the UK due to the introduction of the DTP (diphtheria, tetanus, pertussis)
vaccine. In a child who has moved from outside the UK you should always
consider that immunizations may not have been given. Illness usually begins
with coryza and a dry cough develops after a few days. This cough becomes
more pronounced and occurs in paroxysms. Repeated episodes of coughing
classically end with an inspiratory ‘whoop’. Another name for pertussis is the
‘100-day cough’ as the symptoms often continue for that duration. No treatment
can change the course of the illness, although erythromycin is said to
reduce the period of infectivity. Complications are uncommon but include
pneumonia and bronchiectasis. Immunization does not give total protection,
and protection is not necessarily lifelong.
Bronchiolitis classically presents in the first year of life with a high-pitched cough
and wheeze. In this case you would be right to suspect an inhaled foreign body
though you would usually hear a difference in the air-entry between the left and
right hemithoraces. Also this cough usually has an abrupt onset without the
signs and symptoms of an upper respiratory tract infection.

40
Q

A 7-year-old boy is brought to the emergency department by ambulance following
a 10-minute seizure. He has now stopped fitting but remains drowsy.
He has a 3-year history of headaches, nausea and lethargy which is present
only in the winter months. He lives with his family in a poorly maintained
council flat with gas heating.
Which of the following would confirm your diagnosis?

A. Arterial pO2

B. Carboxyhaemoglobin levels

C. Chest X-ray

D. ECG

E. Oxygen saturations

A

B – Carboxyhaemoglobin levels

Carbon monoxide (CO) can be produced by natural gas combustion devices,
(especially if poorly ventilated), motor vehicle exhausts and burning charcoal
or kerosene. It is therefore more likely that you will come into contact with
CO poisoning in the winter months when heating devices are used. CO has a
very high affinity for binding with haemoglobin (250 times greater than
oxygen), although binding is reversible. Poisoning causes acute symptoms
associated with hypoxia, such as headaches, dizziness and nausea. Cyanosis
does not occur and the skin remains pink. Syncope and seizures can occur
and this may be followed by coma and death.
Blood saturations and blood gases are not useful for diagnosing CO poisoning as
they will not accurately determine the degree of hypoxia. A blood gas may show
metabolic acidosis which corrects on treating the hypoxia. The diagnosis of CO
poisoning is made by taking a carboxyhaemoglobin (COHb) level. Smokers will
have a naturally higher COHb level (up to 9%). A COHb .5% in a non-smoker confirms CO poisoning. You will not be able to gain an accurate result if the
patient has been given 100% oxygen for more than one hour.
Management is with 100% oxygen using a tight fitting mask, which speeds dissociation
of CO from Hb by up to five times. Patients may need intervention for
low blood pressure (fluids) and seizures (anticonvulsants). Hyperbaric oxygen is
required for severe symptoms or if the COHb is .40%.
In this case the source of the CO needs to be found and should be either
removed or fixed. Regular servicing of natural gas burning devices will
prevent many cases of CO poisoning. Household CO detectors are also now
being incorporated into smoke alarms

41
Q

Below is a list of genetic conditions.
Which of the following has an autosomal dominant inheritance pattern?

A. Haemophilia A

B. Incontinentia pigmenti

C. Klinefelter’s syndrome

D. Oculocutaneous albinism

E. Tuberous sclerosis

A

E – Tuberous sclerosis

The inheritance of disorders is a relatively common exam question and needs to
be memorized. Knowing the inheritance of certain disorders is important for
counselling parents regarding the risk of future children being affected.

42
Q

A 12-year-old boy has a prolonged history of atopic eczema. The extensor
surfaces of his limbs are dry, excoriated and inflamed. He says that the
rash is incessantly itchy.
Which of the following would be appropriate adjuncts to his topical emollient
and steroids?

A. Bandages

B. Phototherapy

C. Systemic steroids

D. Tacrolimus

E. All of the above

A

E – All of the above

Atopic eczema (atopic dermatitis) is a chronic inflammatory itchy skin condition
that usually develops in early childhood and follows a remitting and relapsing
course. There is often a breakdown of the skin barrier which makes the skin susceptible to trigger factors, including irritants and allergens. Guidelines on the
management of atopic eczema have been published by NICE1.
When assessing a child, identify potential trigger factors, including irritants
(such as soap and detergents), skin infections, contact allergens, food allergens
and inhaled allergens.
Emollients are the foundation of good eczema care. They should be:
† Used more often and in larger amounts than other treatments
† Used on the whole body even when atopic eczema is clear
† Used while using other treatments
† Used instead of soaps and detergent-based wash products
† Easily available to use at nursery, preschool or schoo

43
Q

A 2-year-old girl is bought into the emergency department by her mother
following a fit. She had been playing with her brother when mum heard
her cry briefly. She then went quiet, looked extremely pale and then fell
to the ground and was unrousable. Mum noticed her body shake for
about 10 seconds before she then came round and was back to her
normal self, playing within minutes. Her brother says she had trapped
her finger in the door just prior to the event.
What is the most likely cause for her loss of consciousness?

A. Absence seizure

B. Breath-holding attack

C. Cardiac arrhythmia

D. Epileptic fit

E. Reflex anoxic seizure

A

E – Reflex anoxic seizure

Whenever a child has a ‘funny turn’ or possible loss of consciousness it is vital to
take a thorough history preferably from the person who witnessed the event.
The parents often think their child has had a fit when in fact there are many
other causes for the loss of consciousness.
This little girl has had a reflex anoxic seizure. These often occur after pain, discomfort
or minor head injuries, with other triggers including fever, cold drinks
or a fright. The child (usually an infant or toddler) becomes very pale and can
fall to the ground. Episodes occur due to a reflex cardiac asystole secondary
to increased vagal response. They can occasionally be associated with tonic–
clonic movements, as in this case, but the child recovers rapidly and is its
usual self soon after the event (unlike after an epileptic seizure).
Breath-holding attacks occur in young children, typically toddlers, and are associated
with the child being upset, angry or crying. The child usually holds their
breath at the end of expiration and can then occasionally go blue and
apnoeic. It is usually seen while the child is screaming. Loss of consciousness
can occur though it is usually only for a brief period and occurs only after
breath-holding has occurred for a while. Simple vasovagal attacks occur when
a child has been standing for long periods or is in a hot stuffy room. The child
will often remember feeling dizzy, lightheaded, experience ringing in their
ears or blurred vision before the syncope. Cardiac arrhythmias can lead to
syncope. These can occur with no warning and may be related to exercise.
Supraventricular tachycardia can lead to syncope though the child usually
experiences palpitations or dizziness prior to the syncope. A prolonged QT
interval, though rare, is a cause of syncope and can be easily ruled out with
an ECG.

44
Q

You see a 3-week-old baby in the community who is being exclusively
breastfed. He has gained weight and is generally well. You have been
asked to see him by the health visitor who is concerned he is yellow. On
examination, you confirm that he is jaundiced but other than this there
is no abnormality to be found.
Which of the following is the most important investigation to perform at
this point?

A. Direct antibody test

B. Split bilirubin

C. TORCH screen

D. Unconjugated bilirubin

E. No investigation required

A

B – Split bilirubin

Any baby who is jaundiced after 14 days (21 days if premature) needs to have a
split bilirubin assay performed. A split bilirubin assay measures conjugated and
total bilirubin levels. If the conjugated fraction is greater than 20% (and .18
mmol/L) then the baby should be seen in a specialist paediatric liver centre.
Examining the stool of a jaundiced neonate is very important as acholic stool
(pale stools due to the absence of bilirubin) suggests an obstruction in the
biliary tree. Detailed investigation is warranted in children with prolonged conjugated
hyperbilirubinaemia. The most important diagnosis to rule out is biliary
atresia. Investigations should also be directed at ruling out infection, metabolic
disorders, hypothyroidism and familial cholestatic syndromes.
The mnemonic TORCH is used to remember the main congenital infections.
T – Toxoplasmosis
O – Other (syphilis)
R – Rubella
C – Cytomegalovirus
H – Herpes
The most common of these is CMV. Features common to all TORCH infections
are jaundice, low birth weight, prematurity, microcephaly, seizures, anaemia,
failure to thrive and encephalitis.

45
Q

A 4-year-old girl presents with a fever, lethargy and vomiting. She complains
of dysuria and has 2þ leucocytes and nitrites on urine dipstick.
There are no signs to find on examination and she is alert and well
hydrated. You make the diagnosis of a urinary tract infection and send
her urine for culture.
What treatment should she receive?

A. A 3-day course of oral antibiotics

B. A 7-day course of oral antibiotics

C. Intravenous antibiotics

D. No treatment required

E. No treatment until microscopy and culture result is obtained

A

A – A 3-day course of oral antibiotics

There are comprehensive NICE guidelines on the treatment of urinary tract
infections (UTIs) in children (www.nice.org.uk:80/nicemedia/pdf/CG54NICE
guideline.pdf).
Infants below 3 months of age should be referred immediately to a paediatric
specialist and treated with intravenous antibiotics in line with treating any feverish
illness in such a young age group.
If the infant or child is older than 3 months and has acute pyelonephritis or
upper urinary tract infection an oral course of antibiotics for 7–10 days
should be used. If oral antibiotics are not tolerated, e.g. excessive vomiting or
lethargy, then intravenous antibiotics should be given initially, followed by
oral antibiotics for a total of 10 days.
If the infant or child is older than 3 months and has a lower urinary tract infection
then a course of oral antibiotics for 3 days is usually adequate. Local guidelines
should be followed for the choice of antibiotics: trimethoprim, co-amoxiclav
and cephalosporins are all suitable. Again, if the child is unable to manage
oral medication then parenteral antibiotics may be used.
If the child is at a high risk of a serious illness or looks seriously unwell, treatment
with parenteral antibiotics is essential.

46
Q

A 4-year-old girl attends the emergency department with a 2-day history of
a palpable purpuric rash over her lower limbs and buttocks. She is systemically
well but recently had a cold.
Which of the following is not a complication of this condition?

A. Abdominal pain

B. Arthritis

C. Conjunctivitis

D. Recurrence

E. Renal failure

A

C – Conjunctivitis

This girl has Henoch–Scho¨nlein purpura (HSP). HSP is an immunologically
mediated diffuse vasculitis, especially of the small blood vessels, often preceded
by an upper respiratory tract infection (especially b-haemolytic streptococci). It
usually occurs between 3 and 10 years of age and is twice as common in boys.
The characteristic finding is a palpable purpuric rash which can occur anywhere,
but is most commonly seen on the buttocks and lower extremities.
The rash in HSP normally resolves after 3 to 4 weeks, but it can come and go for
up to a year after the initial presentation. Other complications of HSP are arthritis
(80%), gastrointestinal involvement (abdominal pain and intussusception) and
renal involvement (glomerulonephritis) leading to acute renal failure (1%). The
prognosis in HSP is very good and most children will have complete resolution
without any serious long-term complications. It is important to check the blood
pressure and dip the urine of every child presenting with HSP.
Treatment is supportive with analgesia for arthritis and abdominal pain. A short
course of systemic steroids can be used in gastrointestinal disease to provide
symptomatic relief of the abdominal pain.

47
Q

A 13-year-old boy has been unwell for a number of days with a sore throat.
Three days after the onset of the illness he developed a widespread rash over
his torso and proximal extremities. The rash is made up of numerous scaly
papules, each 0.5 to 2 cm in size. On examination of his oropharynx you
note bilaterally inflamed tonsils with exudates.
Which of the following is the most likely diagnosis?

A. Atopic eczema

B. Measles

C. Meningococcal sepsis

D. Psoriasis

E. Scarlet fever

A

D – Psoriasis

Guttate psoriasis occurs almost exclusively in children and young adults. The
lesions found are 0.5 to 2 cm oval scaling red papules and small plaques. The
lesions are numerous and distributed over the torso and proximal extremities.
Guttate psoriasis is often preceded by streptococcal infections (as in this case).
The possibility of a concurrent streptococcal infection should be investigated
in any new cases of guttate psoriasis.
Classical psoriasis is a relatively common papulosquamous condition. It is
characterized by well-demarcated, erythematous papules and plaques which
have a scaly surface. Infections, stress, trauma and certain medications may
cause exacerbations.
Atopic eczema can flare up after an acute illness but does not tend to be scaly.
Chronic atopic eczema can have a lichenified appearance and classically affects
the limb extensors rather than the torso. Scarlet fever also presents with a rash
after a group A b-haemolytic streptococcal infection. In scarlet fever the rash
occurs 12–24 hours after the onset of the illness and is a fine diffuse erythematous
maculopapular rash that has the texture of goose flesh. Other features of
scarlet fever include a strawberry tongue, desquamation of the rash after 2 to
3 days (especially of the palms and soles) and lymphadenopathy.

48
Q

You are working in the emergency department when you are called to an
ambulance. A mother has just delivered her baby a few seconds before arriving.
The baby is one minute old with a good cry, is breathing regularly and
has a heart rate of 120/min. She is not limp but has reduced tone. She is pink
centrally but the peripheries are blue.
What is the Apgar score for this baby?

A. 6

B. 7

C. 8

D. 9

E. 10

A

C. 8

Before assessing the Apgar score, the baby should be warmed and dried. The
Apgar is usually performed at 1, 5 and 10 minutes of life. It was devised to
allow a rapid and repeatable assessment of the newborn. The score has a
maximum of 10, with two possible points in five areas of assessment. The
need for intervention actually depends on just three of the five components: respiratory
activity, heart rate and colour

The criteria can be remembered using the mnemonic APGAR:
A Appearance ! colour
P Pulse ! heart rate
G Grimace ! irritability
A Activity ! muscle tone
R Respiration ! respiratory function
49
Q

A 14-year-old girl presents with a one-day history of severe throbbing headache.
She is feeling nauseated but has not vomited. There is no blurring of
vision or flashing lights but she has some photophobia. She had a similar
headache which lasted a day about 3 months ago. She is on the oral contraceptive
pill. Neurological examination is unremarkable and there is no
papilloedema on fundoscopy.
What is the most likely explanation for her headache?

A. Cluster headache

B. Idiopathic intracranial hypertension

C. Intracranial tumour

D. Migraine

E. Tension headache

A

D – Migraine

Headache can be a vague and non-specific symptom in many different diagnoses.
It is therefore important to get an accurate pain history. The mnemonic
SOCRATES can be used to assess any pain including headaches:
S Site
O Onset
C Character
R Radiation
A Associated features
T Timing
E Exacerbating/relieving features; Ever-before
S Severity
Red flag symptoms include an acute-onset of severe pain, pain worse on lying
down, vomiting, developmental regression or personality change, unilateral
pain, papilloedema, hypertension, an increase in head circumference and
focal neurological signs.

Migraines can be unilateral or bilateral. They are often associated with nausea,
vomiting and photophobia. Occasionally migraines are associated with an
aura which can range from flashing lights, black spots to unilateral weakness
or paraesthesia. However these signs are only transient and should disappear
soon after the headache is better. A family history of migraines is often seen
as well as other associated factors such as the oral contraceptive pill and foods
containing tyramine, such as cheese and red wine.
Idiopathic intracranial hypertension (previously known as benign intracranial
hypertension) usually occurs in young women and is associated with obesity.
There is a raised intracranial pressure (ICP) in the absence of a mass lesion.
This can be idiopathic or precipitated by drugs (tetracyclines, oral contraceptive
pill and steroid withdrawal). Features include headache, transient diplopia and
bilateral papilloedema. A CTscan is normal and a lumbar puncture will confirm a
raised cerebrospinal fluid pressure. Management options include weight loss,
avoidance of precipitating drugs and repeated lumbar puncture. The aim of
treatment is to prevent permanent visual loss.
Space occupying lesions are associated with signs and symptoms of raised ICP.
This includes early morning headaches, headaches that are relieved by vomiting,
focal neurological deficits or blurring of vision. There may be evidence of
papilloedema. Hypertension and bradycardia are seen in severe raised ICP.
Tension headaches are often described as a band-like pressure which is precipitated
by stress and is worse late in the day. Cluster headaches are sudden
severe brief attacks of unilateral, periorbital pain associated with eye watering.
They can occur frequently, up to eight times a day, and often at a similar time
of day.

50
Q

A 3-year-old girl is seen in the outpatient clinic due to her longstanding
history of constipation. For the last 6 months she has not passed a
normal stool. She is straining, in pain and only passing very small ‘rabbitlike
droppings’. Her mum is also worried as she is continually soiling her
pants and has not managed to take her out of nappies.
What is the most important initial step in treating this child’s constipation?

A. Encourage increased dietary fibre

B. Enema disimpaction regimen

C. Lactulose twice a day

D. Make sure child is placed on the toilet after every meal

E. Movicol disimpaction regimen

A

E – Movicol disimpaction regimen

This child has chronic constipation and has secondary soiling. Incontinence
occurs in chronic constipation due to the leakage of liquid stool from above
the impacted stool. Until full disimpaction has been achieved, the soiling is
likely to remain. The objectives of treatment are to firstly remove faecal impaction,
restore a bowel habit where soft stools are passed without discomfort, and
then to ensure self-toileting and passing stools in appropriate places (in younger
children). It is essential to establish a good rapport with both parents and the
child as there may be feelings of guilt, blame and shame that all need to be
recognized and dealt with sensitively.
Disimpaction. The objective of disimpaction is to fully clear the rectum of
retained faeces. The use of polyethylene glycol (Movicol) in disimpaction has
been shown to be extremely effective. The disimpaction regime consists of starting
with two sachets of Movicol on day 1 and then increasing the number of
sachets by two every 2 days till a maximum of eight sachets a day. Treatment
should be continued until impaction has resolved or for a maximum of
7 days. Suppositories and rectal enemas can also be used to disimpact but
most paediatricians try and avoid rectal routes unless oral disimpaction has
failed. Very rarely, disimpaction under anaesthetic may be required.

Maintenance therapy. Osmotic laxatives have the best evidence for effectiveness,
and these would include lactulose and Movicol. A randomized controlled trial
showed that Movicol was both more effective and had fewer side-effects than
lactulose. The dose should be adjusted to achieve the regular passage of soft,
formed stools. The chronic use of stimulant laxatives, such as senna, is contentious
though are often used alongside osmotic laxatives. Prolonged use can
cause atonic colon and hypokalaemia. Adequate intake of fluid and fibre
should always be encouraged, and a dietician may be needed.
Behaviour modification. Regular toileting and unhurried time on the toilet should
be encouraged. A reward system, especially one that is geared toward successful
use of the toilet, as opposed to clean pants, is important.
Practice